Download as pdf or txt
Download as pdf or txt
You are on page 1of 28

SURGERY B – Physical Examination of Surgical Patient

Doc Acuna
(PRELIMS)

OBJECTIVES Lipoma
By region
Skin Single cell
Head Defect in lipolysis
Neck Skin glides over the mass
Chest
Breast From doc:
Abdomen Eto ung most common na nakikita sa clinic
Inguinal area and genitalia It is the largest single pathologic cell in the body (single
Anal largest normal cell in the body= ovum)
Pathology Single cell: that very big mass is just ONE CELL
Congenital
Ang problem dito is that yung fat nakakapasok sa
Tumors
adipose tissue tapos hindi a nakakalaba o i doe n
o Benign
o Malignant participate in lipolysis and gluconeogenesis kaya sya
Inflammatory diseases naipon and lumalaki din
Skin glides over the mass: It is a defect in the
subcutaneous tissue; you can pinch the skin over the
SKIN mass
Mass moves with skin Liposuction is a valid option for removal
o Epidermal cysts Usually seen in adults but also has rare cases in infants
o Skin malignancy (congenital lipoma)

Skin glides over the mass


o Lipomas
Neurofibroma
o Soft tissue tumors
o Bone tumors Single or multiple masses
Painless
From doc: Genetic
Madali sa skin masses para malaman if ito ba ay galing Precancerous
sa skin o sa structures underneath the skin (ex: fats,
bones, muscles). From doc:
If you can move the skin over the mass, it is NOT a skin Painless pero lumalaki din kaya lang pag tumubo na sa
appendage tumor. Ibig sabihin hindi sya galing sa skin daliri di na nila maigagalaw young daliri
Genetic: women are usually the carriers (fibroma gene)
& the males are affected
Precancerous: it has to be excised and removed; lahat
Epidermal Cyst
na tutubo sa balat dapat tanggalin
Punctum
Neurofibromatosis
Slowly growing mass
These patches are the precursors of neurofibromatosis-
Sudden enlargement and pain=Infection
cafe a lai o (coffee i h milk)
D ing ge : a ang ago ang i a
From doc:
Just like lipoma, this is not a skin tumor so the incision is
Most common benign skin tumor
LINEAR hindi kailangan na elliptical
U ed o be ebaceo c e ide mal c ince i
no he ebaceo gland ha in ol ed
Punctum: a central, dark comedone opening; makikita Tendon Cyst
mo na may maitim Indications for surgery
Slowly growing: Basically it is a blackhead tapos nagbara o Pain
yung production ng lining epithelium ay maiipon so palaki o >2 cm
sya nang palaki everyday
o Usually na tinitiris ng mga px na may From doc:
mabahong lumalabas Sa litid yan doon ang most common
Management: Antibiotics muna then excise after 2-3 When tendon and tendon sheaths are subjected to work
days para lumiit na kaoonti or irritation
CYST: any encapsulated with a fluid or semi-fluid
substance & kabalikta an n a i olid nod le
Epidermal Cyst Excision, Midback Old treatment: aspirate- pero intact pa din dito ang
o 1st step- Elliptical incision, include the punctum capsule
o If di mo sinami ung punctum & the mass should be As long as the capsule is still intact, it will recur.
removed whole, the cyst will just recur

Page 1 of 9
SURGERY B – Physical Examination of Surgical Patient
Doc Acuna
(PRELIMS)

Pain: this is the area of the ulnar nerve so it has to be Cellulitis


removed kahit na maliit lang na may pain na or malaki >2 o Cellulitis = Streptococcus cellulitis
cm otherwise medical lang ang treatment (massage the Ung 7 groups of antibiotics, any of
area) those ay may effect so kahit alin
At risk: doon gagaling ung patient
o Gamers o Medical yan hindi ito inoopera
o Call center agents Carbuncle
o Bank tellers o Most severe
o Usually found in the back and nape (doon sa
makakapal na skin)
o This CANNOT be treated with incision &
Keloids drainage since it is a very big cavity of necrosis
inside and they are usually all interconnected
Beyond the wound margins
so binibiak sya na malaki (lahat nung nakikita
Medical
na discolored ay tatanggalin) & you let it heal
Avoid surgery
via Secondary intention
From doc:
Collagen type I- has a disarray of collagen.
Dapat may orientation ang hiwa para di sya mag scar Non-Melanoma Skin Cancers
Hypertrophic Scar
Kasi mali ung orientation nung hiwa kasi dapat Basal Cell Carcinoma
PERPENDICULAR to the muscle Squamous Cell Carcinoma
Normal scar is flat; hypertrophic pag elevated ng konti; in
keloids the collagen will go beyond the line of incision. From doc:
2 kinds of skin cancers: Melanoma (colored) & Non-
Those that are of Mediterrenean descent (mga Kastilain) Melanoma (non-colored lesion)
& American-Black are prone to keloids; it is genetic so Basal Cell Carcinoma
namamana sya o The most common & is pinaka mabait din
Keloids o It only needs MINOR surgery with an ellipse
Surgery- you connect the deltoid to the other deltoid, the incision (korteng itlog na hiwa)
front and back o NO chemotherapy & radiotherapy
You do NOT operate on keloids; the more you operate o You will notice that the edges are elevated &
on keloids, the larger they become waxy (waxy edges); parang pearly waxy
Old treatment for keloids- steroids Squamous Cell Carcinoma
Ultimate treatment- pressure (apply pressure, any o It needs surgery
means) o Sometimes chemotherapy but often
Modern treatment- Calcium channel blockers radiotherapy
o According to research, ang problem pala sa o Treatment - madalas ay surgery followed by
keloids ay excessive calcium influx bale ung radiation & then occasionally chemotherapy
collagen kumukuha syang madaming calcium (eto ung malaking difference nya from Basal
o Keloids are assoc with calcium supplements Cell Carcinoma)
so the more calcium that you take in you are
more prone to keloids
Melanoma

Skin Infections Asymmetrical mass


Borders are irregular
Abscess Color variegation
Cellulitis Diameter >6 mm
Carbuncle Elevation

From doc:
From doc:
Abscess is fluctuant (pag hinawakan mo parang may
tubig na laman sa loob) Usually, but NOT ALWAYS, start from moles
o Abscess = aureus (Staphylococcus aureus) If you have a mole with the following characteristics,
Incision & drainage: make a wide enough incision -> let have them biopsied
all of the pus come out (it should be drained out) ABCDE- these are the moles that are prone to develop
o HUWAG/BAWAL nyong pigain ang abscess melanoma
because the area is inflamed at naka open
ung pores ng blood vessels so pede sya
pumunta sa loob ng katawan ng patient

Cellulitis Page 2 of 9
o Cellulitis = Streptococcus cellulitis
Ung 7 groups of antibiotics, any of
those ay may effect so kahit alin
doon gagaling ung patient
o Medical yan hindi ito inoopera
SURGERY B – Physical Examination of Surgical Patient
Doc Acuna
(PRELIMS)

A asymmetrical mass o Lung cancer


B borders are irregular
C color variegation Pag ma b kol a lo, and he kin mo e o e he ma , hen i
D diameter > 6 mm obabl he bone ha in ol ed. I can ei he be an o eoma, o
E elevation metastasis.

Thyroid and lung cancer are the most common cancers that are known
If you have more than 1 color in a for metastasis to the skull. But if metastasis to bone in general,
m le, ha a ici m le. prostate cancer and breast cancer are included.
Ele a i i a bad ig , mea i g i Thi a doc o a ien .
disseminating.

Bleeding is not included

EXERCISE! Identify:

Upon examination, there was thickening of the outer table of skull, skin
glides over the mass, and was thought to be benign. Initial impression
was osteoma.

During surgery, doc cannot remove the mass, and it turned out to be
bone metastasis from thyroid cancer.

Thyroid turned out to have small nodules.

This is a mole with multiple colors, irregular borders, diameter > 6 mm,
and i h ele a ion , o i a Melanoma
Treatment: surgery, chemotherapy, occasionally radiation

Q: What type of thyroid cancer is most commonly involved in bone


metastasis?
Answer: Follicular type

NECK
Location
Borders are elevated, with a necrotic center = Basal Cell Carcinoma Midline
Treatment: surgery alone o Thyroglossal duct cyst vs. goiter
Lateral
o Over the sternocleidomastoid
Branchial cleft cysts (types I to III)
o Under the sternocleidomastoid
Lymph nodes
Level of nodes determines the primary site
o Posterior to the sternocleidomastoid
Lymph nodes
Lymphangioma

In the neck, location is the most important for diagnosis.


Squamous Cell Carcinoma
Treatment: surgery and radiation, occasionally chemotherapy At the exact midline:
If i abo e he h oid ca ilage = thyroglossal duct cyst
* Sa exam doc will describe the lesion and identify. May kasamang If i belo he h oid cartilage = goiter
treatments pero konti lang
Over the SCM:
HEAD May mga butas sa ibabaw ng balat ng SCM, at may lumalabas na
Skull Masses parang sipon, and may makakapa kang parang tracts that are going
Osteoma towards the face = Branchial cleft cysts
o Bone tumor, or from trauma
o Benign If the skin is normal over SCM, and when you palpate
Metastasis under the SCM, and the lymph nodes are enlarged, then it can be
o Thyroid cancer.

Page 3 of 9
SURGERY B – Physical Examination of Surgical Patient
Doc Acuna
(PRELIMS)

Posterior to the SCM: Nodes


At the posterior triangle of the neck, it can be lymph nodes or
lymphangioma. Lymphangioma yung mga bukol na fluctuant na tubig
ang laman, which are benign (old name is cystic hygroma).

Neck Examination
From behind
Swallow
Pulse rate
Stand behind the patient. Using the 2 hands, 3 fingers each, palpate for
the trachea. If you can feel the trachea, there is no goiter.

Level 1 = Submandibular nodes


o Drains the face & anterior 2/3rds of the mouth
Level 2 = Superior jugular nodes
o Drains the tonsils & oropharynx
Level 3 = Middle jugular nodes
o Specific for thyroid cancer
Level 4 = inferior jugular nodes
o Drain the larynx
Level 5 = Posterior triangle nodes
o Drains the nasopharynx
To examine the lymph nodes under the SCM, patient should look to the
Level 6 = Anterior triangle nodes
side and scoop the SCM to put the fingers under the SCM.
o Drains nonspecific structures
A k he a ien o allo . If he ma goe ,i goi e .
There are 6 levels of nodes in the neck. The flow of lymph and venous
The normal pulse rate (since 2012) is 60-90 bpm. If more than 90,
blood is one-way, puro pababa towards the heart.
tachycardia na. If the patient has goiter and tachycardia = toxic goiter
Submandibular nodes (level 1) are the checkpoints for cancer,
Tumors viruses, or microbes. It drains the face and anterior 2/3rds of the
Thyroglossal duct cyst (TGDC) mouth. If you have acne, rotten teeth or gums, all the lymph will go to
Goiter level 1. If level 1 is enlarged, check for pathologies in structures that it
Branchial cleft cyst drains.
If the mass is above the cricoid cartilage = thyroglossal duct cyst
To confirm, ask the patient to pull out the tongue. The mass should go Eyes drain their lymph toward the preauricular lymph nodes (near the
up. tragus of the ear). Afterwards, it will go to level 1 of the neck.

Why is the thyroid gland at the midline? The anlage of the thyroid gland Retroauricular lymph nodes (behind the ear) drains the scalp. It will
is from the base of the tongue and will descend on top of the cricoid, also go to level 1 of the neck after.
and will end at tracheal rings 2,3, & 4.
For superior jugular nodes (level 2), look at the tonsils or oropharynx
If only one side of the thyroid gland enlarges = nodular goiter
If both sides are enlarged = diffuse goiter Middle jugular nodes (level 3) are found at the middle SCM aligned
o he h oid ca ilage. I i a e ecific a ea fo h oid cance . If i
Branchial Cleft Cyst Types enlarged, it can possibly be papillary thyroid cancer.
Type I = if you push the mass over the SCM towards the ear, may
sipon na lalabas sa external auditory meatus Again!
Type II = the primary opening is at the tonsillar pillars Follicular thyroid cancer = metastasizes to the bone
Type III = at the base of SCM, cystic nodule over the SCM. It passes Papillary thyroid cancer = metastasizes to the lymph nodes
through the bifurcation of the carotid arteries, and finally through the
vocal cords Inferior jugular nodes (level 4) drain the larynx.

A he lef ide of he neck, he e the thoracic duct which also drains


the abdomen. If the patient is cachectic and there is an enlarged lymph
node at the left side of the neck, i called he Vi ch de. I a
sign of disseminated pancreatic or gastric cancer

Posterior triangle nodes (level 5) are located behind the SCM. It


drains the nasopharynx. If enlarged, it can be a lymphoma or
nasopharyngeal carcinoma. If a patient has COVID, level 5 enlarges.

Anterior triangle nodes (level 6) are nonspecific in draining


structures.

Thyroid cancer will not involve level 1 & level 2.


Page 4 of 9
SURGERY B – Physical Examination of Surgical Patient
Doc Acuna
(PRELIMS)

Levels 1-4 will not drain to level 5, but to the superior mediastinum Parotid Tumor
(level 7).
Levels 4-6 will drain to the superior mediastinum (level 7).

Thyroglossal Duct Cyst

Branchial Cleft Cyst

Kasama ang cricoid pag inopera


The body of cricoid should go with the specimen otherwise
there will be recurrence

Goiter Mistaken as epidermal cyst because of parang punctum


Presence of mucoid substance

Cervical Nodes

When ome hing ele a e in he neck i ei he achea o


thyroid only
Thyroid cancer is pinaka mabait na cancer because it rarely
80% of of lymph nodes in the neck of Filipinos is tuberculous
metastasize and is known for very slow growth
in origin (no correlation with chest TB)
Lung pathologists are not associated with neck masses
Nodular Goiter Needle biopsy: discharge is parang chalk or matcha yung
color

Lymphoma

Diffuse Toxic Goiter


Tx: medical only (NO surgery)

Diffuse Toxic Goiter Black or maitim yung lymph node = Lymphoma

CHEST
Bony deformities
Symmetrical expansion
Percussion
o Dull
Auscultation
o No breath sounds
Nasopharyngeal cancer
o Transmission of spoken voice
o Ancillary CT scan or ultrasound
Beck T iad
Percussion + Auscultation
Dull plus no breath sounds = Fluid (blood, Pus, Effusion) in the
lungs
Dull with voice transmission = solid/consolidation (pneumonia
or cancer) perform CT scan
Ultrasound is used for fluid in the lungs. If air DO NOT
Check LN level 5
perform ultrasound
Tx: Chemotherapy (secondary) and Radiotherapy (mostly)

Page 5 of 9
SURGERY B – Physical Examination of Surgical Patient
Doc Acuna
(PRELIMS)

Chest Deformity
Breast cancer (stage 3)
Pectus Excavatum
Anaesthesiologist might complain of
difficulty in bagging & ventilation
Notice also that the heart cannot
pump as well because of restriction
Pectus Carinatum
Like chicken breast
Easier ventilation
Non healing wound, painless

Breast non-healing wound = Cancer


Most common mass in women 16-25 years old

Bec T iad Fibroadenoma


1. Neck vein engorgement - hindi
Painless, movable
maka fill up yung lungs leading to
Size not affected by menstruation
venous congestion
2. Muffle heart sounds heart cannot Progressive growth
expand because of fluid in Excision if more than 1cm
pericardium
3. Hypotension Breast Examination
That would be a sign of pericardial effusion In women dapat naka outstretch yung arm
(idk what di nya niliwanag bruh) Best day to examine: first day after menses
Watch the vid if you want to practice ;) (1:02:00)
Fibroadenoma Breast Cyst
Breast mass + Asymmetry Ultrasound: Hypoechoic Not anymore a lesion
Cyst an encapsulated
Painless structure containing fluid or
If o look clo el he e blood in he ni le semifluid substance
NOT a cancer
Ultrasound: Anechoic

Cyst + Solid nodule Complex cyst perform surgery

ABDOMEN
Gynecomastia Inspect
o Globular vs Scaphoid
o The bigger the abdomen the lower the source of the
obstruction
o Globular- Large Bowel Obstruction due to Rectal
Cancer
o Scaphoid
Obstructing Esophageal Cancer
Natural walang air and food nakakarating sa taas
Males have very little amount of estrogen in the body. (may bara / obstruction) so everything below the
Estrogen is excreted in the liver pag di na excrete it will
esophagus will be collapsed.
accumulate leading to enlargement of male breast
Lalagyan ng gastrotomy for food
Tx: replace liver daw

Accessory breast

Palpation for mass


o For pain
o For masses
Responds to the menstrual cycle or breastfeeding o Fluid wave (or Ascites) - That will not happen if walang
if i h m all e i e i i acce b ea ( i h tubig sa Tiyan
without nipple)

Page 6 of 9
SURGERY B – Physical Examination of Surgical Patient
Doc Acuna
(PRELIMS)

inflammed it will not hurt - sasakit lang if namamaga


yung gallbladder sometimes also in liver abscess )
o Rovsing sign
if you press on the left there will be pain on the right.
Kapag pinalpate mo sa kaliwa yung gas doon sa
loob ng colon pupunta sa kanan ; pupunta sa
appendix area
If you PE the abdomen especially in children ang
o Easier to relate it if you separate them in quadrants or huli Mong I-PE yung painful sign. Kasi pag inuna
region mo yung painful side lahat na ng ipalpate mo
o Study the abdominal mass for pain using the quadrants masakit. So the PE will be useless.
or region o McBurney Sign
o 9 regions or quadrants Acute and obviously for appendicitis
If this is a woman the diagnosis will be ectopic
pregnancy, ovarian cyst rupture or appendicitis
But for males we know that there are no other organ
in that area it is Appendicitis

Abdominal Signs (Eponyms)


M h ig
o Respiratory arrest on the subcostal area during deep
breathing
McBurney Sign
Percussion
o Dee ende ne a McB ne oin - sign of acute
appendicitis Auscultate
Rovsing Sign o For bowel sounds the patient should not take meal
o if you press on the left there will be pain on the right. before auscultation
Dunphy sign Place stet in periumbilical area auscultate small
o Increased abdominal pain with coughing. May be an bowels
indicator of appendicitis Normal bowel sounds: 6-11 per minute
o Ask the patient to cough. Pag umubo siya sumasakit <6 : hypoactive
yung tiyan wherever it is. >11 : hyperactive (gastroenteritis)
Blumberg sign If you auscultate area of stomach and colon = <6
o Dinidiinan mo tapos masakit that is direct tenderness but per minute
if you release it then sumakit that is blumberg sign also
known as Rebound tenderness Acute Appendicitis
o *Blumberg and Dunphy sign are both signs of peritonits
Usual age for acute appendicitis is 11-
Psoas Sign 30 years old
o Kinakabit ng psoas muscle yung thigh to the spine. Any
if less than the age bracket usually it is
movement sa thigh at may naka Patong na inflammed sa
not appendicitis
psoas sasakit yung psoas muscle.
e.g. <10y/o at same complaints.
o Most of the time it will be the inflammed appendix
Periumbilical, masakit ang pusod
o When you move your thigh or flex your thigh sumasakit
masakit sa right lower quadrant pero
yung psoas
less than 10 y/o most likely it is
Cullen Sign
Meckel s diverticulitis / diverticulum.
o U = mbilical a ea ; ag ma d go a od ha mean
But if less than 1y/o pwedeng
there is blood inside the abdomen ( if female: possible
intussusception with same complaints
ectopic pregnancy ; male = possible liver or pancreatic
cancer)
Kehr sign Cholecystitis
o Kali a / lef - Left shoulder pain may kasabay na left
upper quadrant pain radiating to the leftshoulder is called Epigastric pain aggravated by eating
kehr sign. Murphy Sign
o Sign of splenic injury, splenic rupture, splenic hematoma
o e.g cyclist who experienced trauma on the left side Hydrops of the Gallbladder
o Ask for CT scan or ultrasound
Ang laman ng gallbladder instead na kulay bile, kulay tubig na
Palpation and that is actually mucin. It is called hydrops of gallbladder
o Murphy sign because before the fluid is mistaken as water that is why it is
Palpating on the subcostal area and ask the patient called hydrops of the gallbladder.
to inhale deeply and there will be respiratory arrest GB palpable beyond the subcostal margin (normally it is not
palpable)
- sumasakit sa cholecystitis and not on
(+)Murphy sign
cholelithiasis (if presence of stones and not
Page 7 of 9
SURGERY B – Physical Examination of Surgical Patient
Doc Acuna
(PRELIMS)

Mirizzi syndrome (important board exam question and will be


given on our exam)
o Jaundice without CBD dilation
When the stone is impacted in the cystic duct, there will be
external compression on the common hepatic duct and the
cbd will seem to be small or normal. The patient is yellowish
but on ultrasound, yes there are stones present at the cystic
duct but the CBD is normal.

Incarcerated or Strangulated Inguinal Hernia


Violaceous large scrotum - umabot sa scrotum so it is Indirect
Inguinal Hernia
Naipit ang bituka
Emergency Operation - open hernia sack and ligaments tapos
makakahinga yan. Notice na hawak yung bowels kasi Baka
Q: Why is the patient yellow? pumasok ulit sa loob kasi naluwagan na. Look for signs of life
A: Due to Mirizzi Syndrome - it will turn to pink if it is without vascular compromise but if it
is strangulated with vascular compromise it will not turn to
pink
Jaundice
Yellow eyes and skin
How does jaundice present?
o Earliest presentation of jaundice will be seen at the
frenulum of the tongue (yellow color) of children
o Usually physiologic jaundice occurs for 1 week - so after
1 wk the child is already yellow but it will disappear after
1 week
o Mild - Frenulum only is yellow
o Moderate - frenulum of tongue up to eyes are yellow Incarcerated incisional Hernia s/p open Appendectomy
o Severe - entire skin and body are yellow
Urine usually is as yellow as the yellow discoloration of the
eyes of patient with jaundice (urine has tinge of green-orange)

Incarcerated incisional Hernia s/p open Appendectomy


The number one predisposing factor for hemorrhoids with
Inguinal Examination for Hernias
women is Pregnancy. Nakadagan yung matress sa
You must be able to differentiate hernia prior to surgery but hemorrhoidal veins galing sa pwet.
intra op there will be no problem. With Males - prolonged sitting in the toilet bowl
All of these three types of hernia will need surgery External hemorrhoids
Requirement for hernia is Reducible Inguinal Mass NOT o Brown
scrotal enlargement o Seldomly operated on since it has presence of clotted
kailangan lumalaki yung singit and reducible (pag humihiga blood only.
nawawala yung bukol - that is a classic reducible inguinal o Sometimes local anesthesia is given and we just slice to
bulge) remove the blood clot I
1. Indirect Inguinal Hernia Internal Hemorrhoids
Younger patient - inborn (with bukol in singit) o Pink
Scrotal enlargement o Grade 1 - bleeding only pure red areterialized blood ; No
mass felt
2. Direct inguinal Hernia o Grade 2 - Mass is felt but when standing it disappears -
usually in elderly ; it does not happen in younger population also called as spontaneous reduction - no operation
because the fascia is strong and intact needed
o Grade 3 - Mass is felt but you have to push it inside to
scrotal enlargement does not occur. There is no way that the
disappear - also called as manual reduction - Operation
sack of direct inguinal hernia will reach the scrotum
is done
o Grade 4 - Mass is felt, violet in color and even if you try
3. Femoral hernia
to push it back inside it will not return - operation is done
woman with subinguinal - ilalim ng subinguinal ligament ang
o *Grade 3 and 4 are the only ones subjected to surgery
ang bukol
o Femoral triangle is under the inguinal ligament
Lumalaki tiayn, sumusuka, no DM = obstruction

Page 8 of 9
SURGERY B – Physical Examination of Surgical Patient
Doc Acuna
(PRELIMS)

Hemorrhoid vs Rectal Prolapse


Rectal Prolapse
o Pag ang lumawit puro circles
Hemorrhoid
o If the furrose is vertical

Perianal Abscess
Victims are usually people who sits for a very long time (seat
heats up and with poor hygiene possiblity of pus formation)
o Medical students, call center agents, bank tellers,
taxi drivers
Fistula-in-Ano
There should no communication between the rectum and the
perianal skin but since there is an abnormal communication
there is a fistula
Pag hindi mo naipasok ng ganyan walang fistula yun. Dapat
may tract na ganyan. Surgeons will only break it apart and it
will heal by itself.

Nec i i g Fa cii i F ie Ga g e e
Complaint of wound in the scrotum
Discharge is blackish and ultra foul smelling
Caused by clostridium sp. anaerobic infection due to a
neglected fistula.

Exam - description of the lesion and what lesion it is

Page 9 of 9
BASIC SURGERY 2A
Acute Abdomen
Rayner M. Baloloy, MD

ACUTE ABDOMINAL PAIN § Surgical Acute Abdomen


§ Pain less than one week in duration - Refers to the existing abdominal pain caused by
§ Severe abdominal pain that is less than 24 hours in diseases that needs surgical treatment
duration - Usually presents with “peritonitis”, peritoneal
§ The patient feel pain anywhere between chest and groin irritation
§ The principal reason for an ED visit in 2011
§ Annual incidence approx. 63/100 ED visits Potential Surgical Nature of the Acute Abdomen
§ Admission rate varies (high as 63% in pts> 65 y/o)
Hemorrhage Perforation
Common Consult in the ED - Solid organ trauma - Perforated
Non-specific abdominal 34% - Leaking or ruptured gastrointestinal ulcer
Appendicitis 28% arterial aneurysm - Perforated
Biliary tract dse 10% - Ruptured ectopic gastrointestinal cancer
SBO/ perforation 4% pregnancy - Boerhaave’s syndrome
GYN dse 4% - Bleeding gastrointestinal - Perforated diverticulum
Pancreatitis 3% diverticulum Blockage
- Arteriovenous - Adhesion induction
Renal colic 3%
malformation of small/large bowel
Perforated ulcer 3% gastrointestinal tract obstruction
Cancer 2% - Intestinal orientation - Sigmoid volvulus
Diverticular dse 2% - Aortoduodenal fistula - Cecal volvulus
Others 6% after aortic vascular graft - Cecal volvulus
- Hemorrhagic pancreatitis - Incarcerated hernias
§ Classification - Mallory-Weiss syndrome - Inflammatory bowel
- You have to classify it first. “Is it a medical acute - Spontaneous rupture of disease
abdomen or a surgical acute abdomen?” Not all acute spleen - Gastrointestinal
abdomen is surgical in nature. malignancy
§ Physiology of Abdominal Pian Infection - Intussusception
§ Diagnosis - Appendicitis Ischemia
- Accurate history and comprehensive physical exam - Cholecystitis - Buerger disease
§ Differential Diagnosis - Meckel’s diverticulitis - Mesenteric
§ Treatment - Diverticular abscess thrombosis/embolism
- Psoas abscess - Ovarian torsion
CLASSIFICATION - Ischemic colitis
Medical Acute Abdomen - Testicular torsion
- Refers to the existing medical disease which can - Strangulated hernias
induce abdominal pain with no surgical or
gynecological indications PHYSIOLOGY OF ABDOMINAL PAIN
- Pain may be alleviated upon control of existing medical Visceral
problem - Due to stretching of fibers innervating the walls of
§ Divided into three categories hollow or solid organs
- Endocrine and metabolic causes - Midline pain due to bilateral innervation, poorly
- Hematologic causes localized to the epigastrium, periumbilical region or
- Toxins and drugs hypogastrium depending on its origin from the
primitive organ, the midgut or hindgut
Nonsurgical Causes of the Acute Abdomen - Steady pian or vague discomfort to excruciating or
Endocrine and Metabolic Causes colicky pain
Uremia - Epigastric
Diabetic crisis - Periumbilical
Addisonian crisis - Suprapubic
Acute intermittent 1porphyria Parietal pain
Hereditary Mediterranean fever - Corresponds to the segmental nerve roots
Hematologic causes innervating the peritoneum
Sickle cell crisis - Tends to be sharper and better localized
Acute leukemia - Tenderness and guarding which progress to rigidity
Other blood dyscrasias and rebound tenderness
Toxins and Drugs - Aggravated by respiration and thoracic movements
Lead Poisoning Referred pain
Other heavy metal poisoning - Is a term used to describe a phenomenon of pain
Narcotic withdrawal perceived at a site adjacent to or at a distance from
Black widow spider poisoning the site of an injury’s origin (i.e., irritation in the
diaphragm may produce a pain in the shoulder)

Ⓙ! 1
- Produces symptoms not signs Associated Nausea and Vomiting
- Based on developmental embryology - Reflex, occurs in many conditions
- Acute appendicitis?
Locations and Causes of Referred Pain - Repeated vomiting of large amounts in gut obstruction, Is
Right Shoulder often bile stained
Liver Diarrhea
Gallbladder - Acute gastroenteritis or food poisoning
Right hemidiaphragm - May also occur in appendicitis or other focal inflammatory
Left Shoulder lesion of the gut
Heart Constipation / obstipation
Tail of pancreas Spleen - With complete SBO – unrelenting constipation
Left hemidiaphragm (obstipation)
Scrotum and Testicles - Progressive constipation with carcinoma of the large bowel
Ureter
Physical Examination
Overall appearance / General survey
- Facial expression, diaphoresis, pallor, and degree of
agitation
- Visceral pain – unable to lie still
- Peritonitis – likely to stay immobile

Inspection
- Address the contour of the abdomen including whether it
appears distended or scaphoid and whether a localized
mass effect is observed
- Scars, hernias, masses
- Evidence of erythema or edema of skin may suggest
cellulitis of the abdominal wall; ecchymosis is sometimes
observed with deeper necrotizing infections of the fascia
or abdominal structures, such as the pancreas.
Auscultation
- can provide useful information about the gastro- intestinal
tract and the vascular system
- Hyperactive BS, hypoactive BS, or absent
- Bowel sounds are typically evaluated for their quantity and
quality
DIAGNOSIS - A quiet abdomen - ileus
§ History - Hyperactive bowel sounds - enteritis and early
§ Physical Examination ischemic intestine
§ Laboratory findings
- The pitch and patterns of the sounds are also considered
§ Imaging Studies
§ Diagnostic Laparoscopy - Mechanical bowel obstruction - high-pitched tinkling
sounds that tend to come in rushes and are
History associated with pain.
§ “Tell me more about your pain…” - Far-away echoing sounds - present when significant
- PQRST of pain luminal distention exists
§ Questions should be open ended whenever possible - Bruits - reflect turbulent blood flow within the
§ Pain identified with one finger is more often localized vascular system.
Percussion
Type of Onset - used to assess for gaseous distention of the bowel, free
- Sudden onset – ruptured viscus, mesenteric thrombosis intra-abdominal air, degree of ascites, or presence of
- Gradual – appendicitis, cholecystitis peritoneal inflammation.
Quality - Hyperresonance (tympany to percussion) - characteristic
- Dull – appendicitis of underlying gas-filled loops of bowel.
- Sharp – renal or obstruction of a gut Palpation
- Stabbing/ piercing – acute pancreatitis - Most critical step
- Pleuritic – intensified by breathing - Typically provides more information than any other
- Tearing – dissecting aneurysm component of the abdominal examination
Frequency and Duration - In addition to revealing the severity and location of
- Transient pain and short duration which does not recur is abdominal pain, palpation can further confirm the presence
usually insignificant of peritonitis and identify organomegaly or an abdominal
- The longer the duration the more likely a surgical condition mass lesion
Factors which intensify or relieve pain - Palpation should always begin gently and away from the
- Relation to meals reported area of pain.
- Posture - Work toward area of pain
- Motion – any movement causes intense pain - WARM HANDS!
- Patient on back, knee bent

Ⓙ! 2
Digital Rectal Examination (DRE)
- Needs to be performed in all patients with acute abdominal ß-HCG
pain, checking for the presence of a mass, pelvic pain, or - Women of childbearing age
intraluminal blood Bilirubin, SGPT, SGOT. Alkaline phosphatase
- A pelvic examination should be included in all women in
evaluating pain located below the umbilicus Imaging Studies
Plain radiographs
Laboratory Findings - Upright chest radiographs can detect as little as 1 mL of air
- Help confirm that inflammation or an infection is present injected into the peritoneal cavity.
and also aid in the elimination of some of the most common - Lateral decubitus abdominal radiographs can also detect
nonsurgical conditions pneumoperitoneum effectively in patients who cannot
stand, 5-10 mL
Helpful Laboratory Studies in the Acute Abdomen - Plain films also show abnormal calcifications.
Hemoglobin Approximately 5% of appendicoliths, 10% of gallstones, and
White blood cell count with differential 90% of renal stones contain sufficient amounts of calcium
Electrolyte, blood urea nitrogen, and creatinine concentrations to be radiopaque.
Urinalysis - Upright and supine abdominal radiographs are helpful in
Urine human chorionic gonadotropin identifying gastric outlet obstruction and obstruction of
Amylase and lipase levels the proximal, mid, or distal small bowel.
Total and direct bilirubin concentration - also aid in determining whether a small bowel obstruction
Alkaline phosphatase is complete or partial by the presence or absence of gas in
Serum aminotransferase the colon.
Serum lactate levels
Stool for ova and parasites
C. difficile culture and toxin assay

Complete blood count - Patients with an acute abdomen


with differential will have either a leukocytosis or
bandemia.
- Assist in evaluating the effect of
Serum electrolyte, such factors as vomiting and third
blood urea nitrogen, space fluid losses.
and creatinine - May suggest an endocrine or
measurements metabolic diagnosis as the cause of
the patient’s problem. Upright chest radiograph depicting moderate-sized
pneumoperitoneum consistent with perforation of abdominal
- May suggest pancreatitis as the viscus.
cause of the abdominal pain, but
Serum amylase and levels can also be elevated in other
lipase determinations disorders, such as small bowel
infarction and duodenal ulcer
perforation. Apex

Liver function tests - Helpful in evaluating potential


(total and direct biliary tract causes of acute
bilirubin, serum amino- abdominal pain.
transferase, and alkaline
phosphatase)
- Helpful in diagnosis of intestinal
Lactate levels and ischemia or infarction.
arterial blood gas Upright abdominal radiograph in a patient with a sigmoid colon volvulus.
Note the characteristic appearance of a bent inner tube with its apex in the
right upper quadrant.
WBC-DC (Differential Counting)
- The total leukocyte count and percentage PMN are usually Abdominal Ultrasonography
elevated in acute inflammatory conditions
- Whereas early in the course of intestinal obstruction, there
- Extremely accurate in detecting gallstones and in assessing
gallbladder wall thickness and the presence of fluid around
may be no significant alterations
the gallbladder
Urinalysis (UA)
- Hematuria suggests disease of the urinary tract - Determining the diameter of the extrahepatic and
- Appendicitis? intrahepatic bile ducts.
- In dehydration of the specific gravity of the urine may be - Detecting common bile duct stones is limited
increased, and the red cell and Hgb values - Abdominal and transvaginal ultrasonography can aid in the
Amylase, Lipase detection of abnormalities of the ovaries, adnexa, and
- Serum amylase in excess of 500 units are significant levels of uterus.
1500 – 2000 units or more are not unusual in the early - Ultrasound can also detect intraperitoneal fluid
stages of severe acute pancreatitis

Ⓙ! 3
- Ultrasound images are more difficult for most surgeons to § A rigid scope is inserted in the umbilicus in an insufflated
interpret than plain radiographs abdomen
§ Working ports are placed in areas of interest

PERFORATED PEPTIC ULCER


§ The three most common complications of PUD, in
decreasing order of frequency
§ Bleeding
§ Perforation
§ Obstruction
§ Perforated peptic ulcer usually presents as an acute
abdomen
- Ultrasonography of the gallbladder § The patient can often give the exact time of the excruciating
- Arrows indicate the acoustic shadows from the stones in abdominal pain
the gallbladder § Initially, a chemical peritonitis develop from a gastric or
duodenal secretion. But within hours, a bacterial peritonitis
prevails. The patient is in obvious distress and the abdominal
Computed Tomography Scan (CT Scan)
examination shows peritoneal signs.
- Widely available and less likely to be hindered by
§ Usually presents with marked involuntary guarding and
abdominal air
rebound tenderness
- A number of studies have demonstrated the accuracy and § Upright chest x-ray shows free air in about 80% of patients
utility of CT of the abdomen and pelvis in the evaluation of § Once the diagnosis has been made, the patient is given
acute abdominal pain analgesia and antibiotics, resuscitated with isotonic fluid, and
- A well-performed CT scan using oral, rectal and IV taken to the operating room
contrast is highly accurate for evaluating abdominal pain § Sometimes, the perforation has sealed spontaneously by the
especially appendicitis time of presentation, and surgery can be avoided if the
- Excellent for differentiating mechanical small bowel patient is doing well
obstruction from paralytic ileus and can usually identify the § Nonoperative management is appropriate only f there is
transition point in mechanical obstruction objective evidence that the leak has sealed (i.e., radiologic
- Some of the most difficult diagnostic dilemmas, including contrast study), and in the absence of clinical peritonitis
acute intesinal ischemia and bowel injury after blunt
abdominal trauma, can often be identified by this method. Treatment
§ Simple patch closure
- Most commonly performed operation for perforated
CT scan of a patient
peptic ulcer
with a partial small
Air fluid level - Procedure of choice in patients with hemodynamic
bowel obstruction.
instability or exudative peritonitis signifying a
Note the presence of
perforation >24 hrs
dilated small bowel and
§ Patch closure and HSV
decompressed small
- In stable patients without longstanding perforation,
bowel.
particularly those with chronic symptoms or failure of
medical treatment, the addition of HSV should be
The decompressed considered.
bowel contains air, § Patch closure with Vagotomy and Drainage
indicating a partial - also an acceptable definitive operation for perforated
obstruction. duodenal ulcer, but occasionally side effects are
disabling, and if gastrojejunostomy has been
performed, marginal ulcer can be life-threatening.

- In the stable patient without multiple operative risk fac-


Pancreatitis tors, perforated gastric ulcers are best treated by distal
- Effusion gastric resection.
- Buildup of fluid - Vagotomy is usually added for type II and III gastric ulcers.
Cholecystitis
- Patch closure with biopsy; or local excision and closure; or
biopsy, closure, truncal vagotomy, and drainage are alter-
native operations in the unstable or high-risk patient, or in
the patient with a perforation in an inopportune location.
- All perforated gastric ulcers, even those in the prepyloric
position, should be biopsied if they are not removed at
Diagnostic Laparoscopy surgery.
§ High sensitivity and specificity
§ Ability to treat a number of the conditions causing an acute - When you do the simple patching, you give the patient lifetime
Abdomen laparoscopically PPI
§ Decreased morbidity and mortality, decreased length of - When you do vagotomy, you do a drainage procedure so the
stay, and decreased overall hospital costs food from the stomach goes directly to the jejunum
§ Diagnostic accuracy is high, and reports show the accuracy
ranges between 90% and 100%, with the primary limitation
being recognition of retroperitoneal processes.

Ⓙ! 4
- Intraperitoneal duodenal perforations require surgical
repair with pyloric exclusion and gastrojejunostomy or tube
duodenostomy.
- Iatrogenic small bowel perforation incurred during
endoscopy, if immediately recognized, can sometimes be
repaired using endoscopic techniques.

Perforation of the jejunum and the ileum


- occurs into the peritoneal cavity and usually causes overt
symptoms and signs, such as abdominal pain, tenderness,
and distention accompanied by fever and tachycardia.

Plain abdominal radiographs - Reveal free intraperitoneal air if


intraperitoneal perforation has occurred.
CT Scan - If perforation is suspected but not clinically obvious

Jejunal and ileal perforations require surgical repair or segmental


resection.
- Jejunorraphy
- Ilieorraphy

Compared to the duodenum, jejunum and ileum presents with


SMALL BOWEL PERFORATION abdominal pain and tenderness
§ Prior to the 1980s, duodenal perforation due to peptic ulcer - As we all know, jejunum and ileum are intraabdominal in
disease is the MC form of small bowel perforation location compared to the duodenum
§ Today, iatrogenic injury incurred during gastrointestinal
endoscopy is the most common cause of small bowel ACUTE APPENDICITIS
perforation Inflammation of the appendix is a significant public health problem
§ Infections (tuberculosis, typhoid, and CMV) - lifetime incidence of 8.6% in men and 6.7% in women
§ Crohn’s disease - highest incidence occurring in the second and third decade
§ Ischemia of life
§ Drugs (e.g., potassium- and NSAID-induced ulcers) The etiology of appendicitis is perhaps due to luminal obstruction
§ Radiation-induced injury - Pediatric populations - occurs as a result of lymphoid
§ Meckel’s and acquired diverticuli
hyperplasia in
§ Neoplasms (Lymphoma, adenocarcinoma, and
- Adults - may be due to fecaliths, fibrosis, foreign bodies
melanoma)
(food, parasites, calculi), or neoplasia
§ Foreign bodies
Obstruction ➡ increase intraluminal pressure ➡ impaired venous
§ Among iatrogenic injuries, duodenal perforation during
drainage ➡ mucosal ischemia ➡ bacterial translocation ➡ gangrene
endoscopic retrograde cholangiography (ERCP) with
endoscopic sphincterotomy (ES) is the most common.
- Incidence of this is decreasing but remains at Escherichia coli and Bacteroides fragilis
around 0.5%. - The most common aerobic and anaerobic bacteria
isolated in perforated appendicitis.
The Stapfer classification
- is commonly used to categorize different types of History
ERCP-related perforations. These are: - It is important to elicit an accurate history from the patient
Type I: Free bowel wall perforation and/ or family, in the case of pediatric patients.
Type II: Retroperitoneal duodenal perforation secondary to - Inflammation of the visceral peritoneum usually progresses
periampullary injury to the parietal peritoneum, presenting with migratory pain,
Type III: Perforation of the pancreatic or bile duct Type IV: which is a classic sign of appendicitis
Retroperitoneal air alone - Inflammation ➡ anorexia, nausea, vomiting, and fever.
*Type II (retroperitoneal duodenal injuries) are the most common and - Regional inflammation ➡ ileus, diarrhea, small bowel
can often be managed nonsurgically. obstruction, and hematuria.

Physical Examination
Diagnostics
- Most patients lay quite still due to parietal peritonitis.
- CT scanning is the most sensitive test for diagnosing
- Patients are generally warm to the touch (with a low-grade
duodenal perforations
fever
- Positive findings include pneumoperitoneum for free
- Demonstrate focal tenderness with guarding
perforations, but more commonly
- retroperitoneal air,
- One-third of the distance
- contrast extravasation, and
between the anterior
- paraduodenal fluid collections.
McBurney’s point superior iliac spine and the
umbilicus
Management
- Point of maximal
- Retroperitoneal perforations of the duodenum can be
tenderness in a patient
managed nonoperatively in the absence of progression and
sepsis.

Ⓙ! 5
Pain in the right lower Magnetic resonance Imaging
quadrant after release of gentle - Sensitivity of 0.95
Rovsing’s sign pressure on left lower - Expensive test
quadrant - Requires significant expertise

Pain with coughing (retrocecal Acute appendicitis is a clinical diagnosis


Dunphy’s sign appendix) - By history and PE and laboratory alone, you can diagnose
acute appendicitis
Pain with internal rotation of - Imaging is not always needed
Obturator sign the hip (pelvic appendix)

Pain with flexion of the hip Differential Diagnosis


Iliopsoas sign (retrocecal appendix) § Acute mesenteric adenitis
§ Cecal diverticulitis, Meckel’s diverticulitis
§ Acute ileitis, Crohn’s disease
§ Acute pelvic inflammatory disease
Laboratory Findings § Torsion of ovarian cyst or graafian follicle
§ Leukocytosis - 10,000 cells/mm3 § Acute gastroenteritis
§ Gangrenous and perforated appendicitis (∼17,000 § Mittelschmerz
cells/mm3) § Ectopic pregnancies
§ C-reactive protein
§ Bilirubin, Il-6 Management
§ Procalcitonin Uncomplicated Appendicitis
*White blood cell (WBC) count and a C-reactive protein - § Appendectomy
are two appropriate lab tests to obtain in the initial work up § Nonoperative management
of appendicitis - 26.5% of patients in the nonoperative group
§ Pregnancy test - also essential in women of childbearing age required an appendectomy within 1 year.
§ Urinalysis - valuable in ruling out nephrolithiasis or Timing of surgery
pyelonephritis. § Emergent surgery
§ Urgent surgery (waiting less than 12 hours)
Imaging Approach of surgery
§ We request for imaging if we have equivocal findings § Laparoscopic appendectomy
(history and PE does not jive) § Open appendectomy
§ utilized to confirm a diagnosis of appendicitis because a Complicated Appendicitis
negative operation rate is acceptable in <10% of male - Perforated and gangrenous appendicitis and
patients and <20% of female patients. appendicitis with abscess or phlegmon formation
§ Are most appropriate for patients in whom a diagnosis of § Resuscitation
appendicitis is unclear or who are at high risk from § IV antibiotics
operative intervention and general anesthesia, such as § Appendectomy
pregnant patients or patients with multiple comorbidities. § Nonoperatively

Computerized tomography (CT)


- Sensitivity and specificity of 0.96 Operative Intervention
- Features on a CT scan that suggest appendicitis § Once the decision to proceed with surgical intervention is
- Enlarged lumen made, patients can be taken to the operating room rather
- Wall thickness (Greater than 6mm) expeditiously.
- Wall thickening (Greater than 2mm) § While resuscitative efforts are important in patients who
- Periappendiceal fat stranding present with significant dehydration or in a compromised
- Appendicolith host, the majority of patients can be taken to the operating
- For adults room within a short interval.
§ Placement of a Foley catheter is optional but not necessary
Ultrasound (US) while performing an appendectomy.
- Sensitivity of 0.85
- Graded compression ultrasonography - used to identify the § Preoperative antibiotics must be administered at least 30
anteroposterior diameter of the appendix to 60 minutes prior to skin incision.
- Easily compressible appendix <5mm in diameter generally § Cefoxitin, Ampicillin/sulbactam, and cefazolin plus
rules our appendicitis metronidazole
- Features on an ultrasound that suggest appendicitis § Patients with ß-lactam allergies can be given
- Diameter of greater than 6 mm clindamycin in combination with a fluoroquinolone,
- Pain with compression gentamicin, or aztreonam.
- Presence of an appendicolith
Operative Technique
- Increased echogenicity of the fat Open Appendectomy
- Periappendiceal fluid - McBurney’s incision
- For pediatric patients - Rocky Davis incision
- Midline laparotomy
Laparoscopic Appendectomy

Ⓙ! 6
Novel Techniques - Treated in the hospital with parenteral antibiotics
- Single incision appendectomy and bowel rest
- Natural orifice transluminal endoscopic surgery (NOTES) - Most patients improve within 48 to 72 hours. Failure to
- Robotic appendectomy improve may suggest abscess formation.
- CT Scan
Incidental Appendectomy - Deterioration in a patient’s clinical condition and the
development of peritonitis are indications for laparotomy.
COLONIC DIVERTICULITIS
§ Outpouching of the wall of digestive system most commonly Many surgeons now will not advise colectomy even after 2
seen in the colon documented episodes of diverticulitis assuming that the px is
completely asymptomatic and the carcinoma has been excluded
Diverticular Symptomatic diverticula by colonoscopy except for immunosuppressed patients (still
advised to undergo colectomy after a single episode of
disease
diverticulitis).
Diverticulosis Diverticula without inflammation
Diverticulitis Inflammation and infection associated with
diverticula Complicated Diverticulitis
- Diverticulitis with abscess, obstruction, diffuse peritonitis
False diverticula - Pulsion diverticula (free perforation), or fistulas between the colon and
- Majority of the colonic diverticula adjacent structures
- In which the mucosa and muscularis mucosa - Colovesical, colovaginal. and coloenteric
have herniated through the colonic wall.
- These diverticula occur between the Hinchey Staging System
taeniae coli, at points where nutrient - is often used to describe the severity of complicated
arterial blood vessels penetrate the colonic diverticulitis
wall (presumably creating an area of relative Stage I colonic inflammation with an
weakness in the colonic muscle). associated pericolic abscess
- Resulting from high intraluminal pressure Stage II colonic inflammation with a
retroperitoneal or pelvic abscess
True diverticula - Compromise all layers of the bowel wall Stage III purulent peritonitis
- Congenital in origin Stage IV fecal peritonitis

Treatment
- Depends on the patient’s overall clinical condition and the
§ Diverticular bleeding can be massive but usually is self- degree of peritoneal contamination and infection
limited. - Small abscesses (<2cm) – parenteral antibiotics
§ Sigmoid colon is the most common site of diverticulosis - Large abscesses – CT-guided percutaneous drainage and
antibiotics
§ Acquired disorder
- Hinchey Stages I & II – Sigmoid colectomy with primary
§ Lack of dietary fiber ➡ smaller stool volume ➡ requiring anastomosis
high intraluminal pressure and high colonic wall tension for - Larger abscesses, peritoneal soiling, or peritonitis – sigmoid
propulsion colectomy with end colostomy and Hartmann’s pouch
- Success also has been reported after sigmoid colectomy,
Inflammatory complications (Diverticulitis) primary anastomosis and proximal diversion (loop
- Inflammation and infection associated with a diverticulum ileostomy)
- Occur in 10% to 25% of people with diverticulosis - Laparoscopic lavage and drainage without bowel resection
- The spectrum of disease ranges from mild, uncomplicated may be safe and effective even in the presence of free
diverticulitis to free perforation and diffuse peritonitis perforation
- Left sided abdominal pain, with or without fever and
leukocytosis
- Plain radiographs References
§ Dr. Baloloy’s ppt & zoom lecture (2021)
- CT Scan § Schwartz Principles of Surgery (11th Edition)
§ Sabiton Textbook of Surgery (20th Edition)

Differential Diagnosis
§ Malignancy
§ Ischemic colitis
§ Infectious colitis
§ Inflammatory bowel disease

Uncomplicated Diverticulitis
- Left lower quadrant pain and tenderness
- CT findings include pericolic soft tissue stranding, colonic
wall thickening, and/or phlegmon
- TX – Outpatient therapy with broad spectrum
- Oral antibiotics and a low-residue diet

Ⓙ! 7
Ⓙ! 8
BASIC SURGERY 2A
Intestinal Obstruction
Roman P. Oblena Jr., MD

EPIDEMIOLOGY
The accumulation of gas usually comes from swallowed air and the fluid
§ Mechanical SBO – most frequent surgical disorder of the small
consist of swallowed liquids and GIT secretions like saliva ➡ as the gas
intestine
Obstructing lesion and anatomical relationship and the fluid accumulates, there will be bowel distention ➡ intraluminal
§ Intraluminal – foreign bodies, gallstones, meconium and intramural pressure will increase ➡ intestinal motility will decrease,
§ Intramural – tumor, Crohn’s disease-associated fewer contractions
inflammatory
§ Extrinsic – adhesions, hernias, carcinomas CLINICAL PRESENTATION
4 Cardinal Features
§ Intra-abdominal adhesions related prior to abdominal surgery - Order and degree of manifestation depends on the level of
account for 70% of SBO obstruction
§ Cancer-related SBO are commonly due to extrinsic Abdominal pain
compression or invasion by advanced malignancies arising from - First feature
other organs - Severe and sudden onset
- If constant and/or localized, suggests impending bowel
Small bowel obstruction: common etiologies compromise from ischemia and or perforation
Adhesions Vomiting
Neoplasms - Onset: proximal obstruction (short interval between
Primary small bowel neoplasms onset of pain and vomiting); colonic obstruction (late or
Secondary small bowel cancer (e.g., melanoma- absent)
derived metastasis) - Nature: gastric outlet obstruction (undigested stomach
Local invasion by intra-abdominal malignancy contents without bile); distal (bilious becoming more
(e.g., Desmoid tumors) faeculent as obstruction moves distally)
Carcinomatosis Distention
Hernias - Greater degree if more distal obstruction and longer
External (e.g., inguinal and femoral) episode
Internal (e.g., following Roux-en-Y gastric bypass Absolute
surgery) - Failure to pass stool or flatus; occurs earlier in distal
Crohn’s disease obstruction; passage of flatus and stool 6-12 hours
Volvulus - (partial obstruction
Intussusception
Radiation-induced stricture Other signs and symptoms
Postischemic stricture Bowel Sounds
Foreign body
- Initially hyperactive, minimal in late stages
Gallstone ileus
Laboratory findings
Diverticulitis
Meckel’s diverticulum - Reflect intravascular volume depletion
Hematoma - (hemoconcentration and electrolyte abnormalities); mild
Congenital abnormalities (e.g., webs, duplications, leukocytosis
and malrotation) Strangulated obstruction (red flags for surgical intervention)
Congenital causes of SBO - Abdominal pain disproportionate to the degree of
§ Evident during childhood abdominal findings
§ May be diagnosed during adulthood presenting with - Suggestive of intestinal ischemia; tachycardia, localized
abdominal symptoms abdominal tenderness, fever, marked leukocytosis and
Intestinal malrotation and midgut volvulus acidosis
§ Considered in adults with acute or chronic symptoms of
SBO (especially among those with no prior abdominal DIAGNOSIS
surgery) Goals:
Rare etiology of SBO a) Distinguish mechanical obstruction from ileus
§ Superior Mesenteric Syndrome: compression of the 3rd b) Determine the etiology of the obstruction
portion of the duodenum by SMA c) Discriminate partial from complete obstruction
d) Discriminate simple from strangulating obstruction
PATHOPHYSIOLOGY
§ Strangulated bowel obstruction History
§ Partial small bowel obstruction § Ask if there are
- Narrow lumen but allows transit of some content - Prior abdominal operations – suggesting presence of
§ Complete small bowel obstruction adhesions
- Total lumen obstruction - Presence of abdominal disorders (malignancy or IBD)
§ Closed loop obstruction Examination
- Both ends are obstructed i.e., volvulus § Check for possible presence of hernia
Diagnosis
§ Usually confirmed with radiographic examination

Ⓙ! 1
RADIOGRAPHIC EXAMINATION (ABDOMINAL SERIES) TREATMENT
a) A radiograph of the abdomen with the patient in a supine Fluid resuscitation
position - Using isotonic fluid given intravenously
b) A radiograph of the abdomen with the patient in an upright
position Indwelling bladder catheter
c) A radiograph of the chest with the patient in an upright - To monitor urine output
position
Central venous or pulmonary-artery catheter monitoring
Triad of SBO - Not generally indicated unless the patient has underlying
§ Dilated small bowel loops (>3 cm in diameter) cardiac disease and severe dehydration
§ Air-fluid levels seen on upright films
§ A paucity of air in the colon Broad-spectrum antibiotics
- Not indicated unless there is concern for bowel ischemia and
surgery is planned

NG tube replacement
- For continuous evacuation of air and fluid in the stomach
(decreases nausea, distention, and the risk of vomiting and
aspiration)

Expeditious surgery
- Mainstay of treatment for partial bowel obstruction
- Minimizes the risk for bowel strangulation
- GOAL: operate before the onset of irreversible ischemia

Conservative therapy
- NGT and fluid resuscitation
- Commonly recommended in the initial management of
nonischemic partial SBO
Computed Tomography
§ Ideally done with contrast Elective surgery
- Usually performed after administration of oral eater- - If symptoms do not improve within 48 hours after initiation of
soluble contrast or dilated barium nonoperative therapy
- Water-soluble contrast – prognostic and therapeutic
(w/in 24 hours of administration, predictive of Water-soluble oral contrast
nonsurgical resolution of bowel obstruction with a - Diagnostic, prognostic and therapeutic
sensitivity of 92% and a specificity of 93%)
§ Provides global evaluation of the abdomen (reveal the etiology
Operative procedures
of the obstruction)
- According to etiology
- Important in the acute setting when intestinal
- Adhesions are lysed
obstruction represents only one of many diagnoses in
- Tumors are resected
patients presenting with acute abdominal conditions
- Hernias are reduced and repaired
§ Sensitivity (80% to 90%) and Specificity (70% to 90%)
§ Limitations Note: regardless of the etiology, affected intestine should be examined
- Low sensitivity (<50%) in the detection of low-grade
and non-viable bowel should be resected
or partial SBO
§ May provide evidence of closed loop obstruction or Assessment for viability: (visual inspection of Doppler technique)
strangulation - Color
§
- Peristalsis
Closed-loop obstruction - Marginal arterial pulsations
- U-shaped or C-shaped
dilated bowel loop (with a
radial distribution of Decision point in a hemodynamically stable patient
mesenteric vessels § Short lengths
converging toward a torsion - Should be resected
point) - Primary anastomosis of the remaining intestine should
be performed
Strangulation § Larger proportion involved
- Thickening of the bowel wall - concerted effort to preserve intestinal tissue should
- Pneumatosis (Air in the be made (bowel of uncertain viability should be left
bowel wall) intact; reexplore in 24-48 hours)
- Portal venous gas
- Mesenteric haziness
- Poor uptake of intravenous
contrast into the wall

Ⓙ! 2
TREATMENT
1) Laparoscopic adhesiolysis to check for viability by inspecting
color, peristalsis and marginal arterial pulsations. Doppler
probe may also be used
2) Fluid resuscitation to monitor urine output
3) Broad spectrum antibiotics
4) Nasogastric tube for continuous gastric decompression

Perioperative mortality rate for Perioperative mortality rate for


NON-STRANGULATING SBO STRANGULATING SBO
<5% mortality rate Higher mortality rates
Most deaths occurring in elderly Needs prompt intervention
patients with significant
comorbidities

<20% Readmission over the subsequent 5 years with


another episode of bowel obstruction
5.5% at 1 year Risk of recurrent obstruction
11.3 at 3 years
3.5 at 5 years
3.7 at 1 year Risk of reoperation for recurrent obstruction
4.8 at 3 years
5.8 at 5 years

OUTCOMES
LAPAROSCOPIC SURGERY § Long-term prognosis
- Related to etiology of obstruction
Advantages Disadvantages § Many patients treated conservatively for adhesive SBO do not
Significantly lower rates of Iatrogenic bowel injury require future readmissions
overall complications § Standard hospital-wide policy can help improve care of patients
Less surgical site infections Greater surgical time with bowel obstruction, reducing their time to surgery and
Shorter length of hospital stay (4 shortening their length of hospital stay
Vs. 10 days)
- Early cases of proximal small bowel obstruction that are likely due PREVENTION
to a single adhesive band are best suited for this approach § Cornerstone of adhesion prevention
- Good surgical technique
- Careful handling of tissue
- Minimal use and exposure of peritoneum to foreign
bodies
§ Use of laparoscopic surgery, when possible
- Open surgery – associated with a fourfold increase in
risk of small bowel obstruction within 5 years of the
index procedure
§ Adhesion prevention therapy
- Use of hyaluronan-based agents (e.g. Sperafilm)
reduces the incidence of postoperative bowel
Intestinal obstruction secondary to postoperative adhesions adhesions

DIAGNOSIS: COMPUTED TOMOGRAPHY GASTRIC OUTLET OBSTRUCTION


§ Potion of the pylorus
Presentation
- Nonbilious vomiting
- Profound hypokalemic, hypochloremic, metabolic
alkalosis
- Dehydration
- Pain or discomfort
- Prominent weight loss
Initial treatment
- Nasogastric suction
- IV hydration
- Electrolyte repletion
- Acid suppression
- Discrete transition zone with dilation of bowel proximally Most patients with obstructing ulcer disease require intervention
- Decompression of bowel distally - Balloon dilation or operation
- Intraluminal contrast that does not pass beyond the transition zone Clinical findings
- Colon containing little gas or fluid - Succussion splash may be audible with stethoscope
plsced in the epigastrium

Ⓙ! 3
TREATMENT: COLORECTAL CANCER
Diagnosis § Objectives: To remove primary tumor along with its
- Confirmed by endoscopy lymphovascular supply
- Length of the bowel resected depends on which
LARGE BOWEL OBSTRUCTION SECONDARY TO COLON RECTAL vessels are supplying the segment involved
Colorectal carcinoma § Any adjacent organ or tissue should be resected
§ Most common malignancy of the gastrointestinal tract § If all of the tumor cannot be removed
§ Incidence: similar in men and women - PALLIATIVE PROCEDURE
§ Risk factors:
- Aging: increased risk >50 years old § Subtotal or Total Colectomy
- Hereditary: 20% arise in patients with a known family - Presence of synchronous cancer/ adenomas/ strong
history of colorectal cancer family history
- Environmental and dietary: high animal and low fiber - At risk for carcinoma
diet
- Long-standing Inflammatory Bowel Disease CONDITION SURGICAL PLAN
Carcinomatosis (Non-obstructing)Chemotherapy
Polyps Carcinomatosis (Obstructed or - Palliative resection
§ A nonspecific clinical term that describes any projection from imminent obstruction) - Diverting ostomy
the surface of the intestinal mucosa regardless of its histologic - Bypasss procedure
nature - Stent
§ Types
- Neoplastic (tubular adenoma, villous adenoma, OBSTRUCTION SECONDARY TO INCARCERATED HERNIA
tubulovillous adenoma, serrated adenomas/polyps) § Incarcerated hernia should be suspected with any of the
- Hyperplastic following clinical manifestations:
- Hamartomatous (juvenile, Peutz-Jeghers, Cronkite- - Severe abdominal pain, with persistent pain during
Canada) the interim periods of paroxysmal pain
- Inflammatory (pseudopolyp, benign, lymphoid polyp) *Don’t forget to examine groin / scrotal area
- Gradually increased shock
Neoplastic polyps - Evident peritoneal irritation, and increased body
- Adenomatous polyps are common temperature, pulse rate and WBC count
- > 50 years old - Bloody fluid in vomit or intestinal excreta, or from
- Characteristic: dysplastic lesions abdominal puncture asymmetrical bloating, palpable
- Morphology: and tender intestinal loops with rebound tenderness
Pedunculated – mushroom-like on a stalk of Diagnosis
submucosa; Amenable to colonoscopic snare excision
Sessile – broader base; special colonoscopic Xray KUB
techniques (saline lift, piecemetal snare excision, endoscopic
mucosal resection) - Bowel dilatation, multiple air-fluid levels
Hyperplastic Polyps or other characteristic findings related to
- Extremely common in the colon intestinal obstruction
- Usually small (<5mm)
- Characteristic: hyperplasia without dysplasia
- Not considered premalignant
- Cannot be distinguished from adenomatous polyps
colonoscopically
- Often removed
- Large hyperplastic polyps (>2cm): risk of malignant degeneration Ultrasound examination
Serrated Polyps - Expansion of the intestines with
- Endoscopically: flat lesions, difficult to visualize reverse peristalsis, or fixed masses
- Similar to hyperplastic polyps with minimal without peristalsis, or expansion of a
malignant potential fluid-filled bowel
- Treated similarly with adenomatous polyps - Intestinal fluid reflux and thickening
Hamartomatous Polyps (Juvenile Polyps) and edema of the intestinal wall can be
- Usually not premalignancy observed as well as slightly echogenic,
- May occur at any age long strip-shaped omentum in the hernia
- Common symptom: bleeding sac
- Other symptoms: intussusception, obstruction - Observation of the blood supply in the hernia contents with
- Treatment: Polypectomy color Doppler ultrasound
Inflammatory Polyps (Pseudopolyps) CT Scan
- Occur most commonly with IBD - Bowel dilatation, mesangial thickening
- Not premalignant - Following oral iodinated contrast: to
- Microscopic examination: islands of normal determine if the contents in the sac are
- regenerating mucosa surrounded by areas of intestinal
mucosal loss - Enhanced scans can help identify the
presence of bowel strangulation

Ⓙ! 4
Management
§ Any type of hernia can stragulate or cause bowel obstruction
§ Incarcerated hernia (non-reducible)
- Surgical emergency – compromise of blood supply ➡
ischemia ➡ infarction and perforation
§ Incarceration
- Common as up to 20% if patients present with a non-
reducible hernia
- Either open or laparoscopic repair can be performed
➡ approximately the medial the lateral edges of
transversalis fascia to the rectus sheath

NYHUS classification (Inguinal Hernia)


Type I Indirect hernia; internal Excision of hernial sac
abdominal ring normal only
Type II Indirect hernia; internal Herniotomy
ring enlarge without Tighten deep inguinal
impingement on the ring
floow of inguinal canal
Type IIIA Direct hernia
Type IIIB Indirect hernia; indirect Herniotomy
sliding or scrotal Reinforcement of
posterior inguinal wall
Type IIIC Femoral hernia Herniotomy
Tighten femoral ring
Type IV Recurrent hernia Herniotomy and
reinforcement

References:
§ Dr. Oblena’s zoom lecture and ppt 2021

p.s. this is based on the ppt only hehe use at your own risk J

Ⓙ! 5
FEU – NICANOR REYES MEDICAL FOUNDATION
SURGERY B GIT BLEEDING
Michael L. Capulong, MD, FPCS, FPSGS
ACUTE GI BLEEDING
- Acute gastrointestinal (GI) hemorrhage is a common clinical problem
with diverse manifestations.
- The bleeding may range from trivial to massive and can originate from
almost any region of the GI tract, including the pancreas, liver, and
biliary tree.
- Hemorrhage can originate from any region of the GI tract and is
typically based on the location relative to the Ligament of Trietz.
UPPER GI BLEEDING (UGIB) • Peptic Ulcer Disease
proximal to the ligament of Trietz • Variceal Bleeding
LOWER GI BLEEDING (LGIB) • Diverticula Angiodysplasia
colon (more common: SI)
- 85%: cases stop bleeding spontaneously
- 15% cases: require emergent resuscitation, evaluation, and treatments;
usually elderly patients or with comorbidities

GIT BLEEDING
• In patients with GI bleeding,
several fundamental
principles of initial
evaluation and management
must be followed.
• Rapid initial assessment
permits a determination of
the urgency of the situation.
• Resuscitation is initiated with • What will be the guide in identifying the area of bleed?
stabilization of the patient’s o The presence and absence of fresh blood in the stool i.e.
hemodynamic status and the hematochezia, melena, coffee ground, etc.
establishment of a means for • If there is UGIB, do endoscopy within 24 hours. It can be diagnostic or
monitoring ongoing blood non-diagnostic.
loss. o Diagnostic: be able to see the area of bleeding
• A careful history and o Therapeutic: be able to cauterize/clip the bleed and inject
physical examination should epinephrine to stop the bleeding
provide clues to the cause o Non-diagnostic: slow hemorrhage à RBC scan/tagging to
and source of the bleeding pinpoint the exact location of the bleed or massive hemorrhage
and identify any ® angiography operation
complicating conditions or • For LGIB, mainstay would be colonoscopy. It is the same as UGIB in
medications. Specific terms of diagnostic/non-diagnostic.
investigation should then proceed to refine the diagnosis.
• Therapeutic measures are then initiated, and bleeding is controlled ACUTE UPPER GASTROINTESTINAL HEMORRHAGE
and recurrent hemorrhage prevented. • Upper GI refers to the bleeding that arises from the GI tract proximal
to the ligament of Trietz and accounts for nearly 80% of significant
GI hemorrhage.
• The causes of upper GI bleeding are best categorized as either
nonvariceal sources or bleeding related to portal hypertension.
• The nonvariceal causes account for approximately 80% of such
bleeding, with PUD being the most common.
• Although patients with cirrhosis are at high risk for development of
variceal bleeding, nonvariceal sources can account for up to 50% of
GI bleeds.
• However, because of greater morbidity and mortality of variceal
bleeding, patients with cirrhosis should generally be assumed to have
variceal bleeding and appropriate therapy initiated until an emergent
endoscopy has demonstrated another cause for the hemorrhage.
• The foundation for the diagnosis and management of patients with
an upper GI bleed is an upper endoscopy.
Algorithm for the diagnosis of acute GI hemorrhage.
• Subsequent evaluation depends on the results of the upper
endoscopy and the magnitude of the bleeding. Angiography or even ACUTE LOWER GASTROINTESTINAL HEMORRHAGE
surgery may prove necessary for massive hemorrhage, precluding • The mortality rate of lower GI bleeding is similar to that of upper
endoscopy, from either the upper or lower GI tract. GI bleeding at around 3%, but this rate increases with age to
• For slow or intermittent bleeding from the lower GI tract, colonoscopy more than 5% in those 85 years or older.
is now the initial diagnostic maneuver of choice. When endoscopy is • In more than 95% of patients with lower GI bleeding, the source
non-diagnostic, the tagged RBC scan is usually employed. of hemorrhage is the colon.
• For obscure bleeding, usually from the small bowel, the capsule • The small intestine is only occasionally responsible, and because
endoscopy is becoming the appropriate study. Once the bleeding has these lesions are not typically diagnosed with the combination of
been identified, appropriate therapy can be initiated. upper and lower endoscopy, they are considered in the section
on obscure causes of GI bleeding.
• In general, the incidence of lower GI bleeding increases with
age, and the cause is often age related.

1⏐6
liv
DIFFERENTIAL DIAGNOSIS OF LOWER GASTROINTESTINAL - Bleeding stops spontaneously
SURGERY B
HEMORRHAGE - Upper endoscopy confirms the suspicion à one or more longitudinal
COLONIC BLEEDING 90% SMALL BOWEL BLEEDING 5% fissures in the mucosa of the herniated stomach as the source of
Diverticular Disease 30-40% Angiodysplasias bleeding
Anorectal Disease 5-15% Erosions or ulcers (potassium, - Controlled by balloon tamponade
Ischemia 5-10% NSAIDS) - Vasopressin infusion
Neoplasia 5-10% Crohn’s disease - Endoscopic injection of epinephrine may be therapeutic if bleeding
Infectious Colitis 3-8% Radiation does not stop spontaneously
Post-polypectomy 3-7% Meckel’s diverticulum - Surgical: laparotomy and high gastrotomy with oversewing of the
Inflammatory Bowel Disease 3-4% Neoplasia linear tear
Angiodysplasia 3% Aortoenteric fistula
Radiation colitis or proctitis. 1-3%
Other 1-5% BLEEDING PEPTIC ULCER
Unknown 10-25% - Bleeding is the most common cause of ulcer-related death
- MOST COMMON cause of Upper GI Bleeding 75% of patients will
ENDOSCOPY stop bleeding
Algorithm for the diagnosis and management of nonvariceal upper GI - 25% continue to bleed or rebleed (Mortalities occur In this group)
bleeding. - Early endoscopy needed to diagnose and assess any Hemodynamic
• As stated previously, therapy
patients with clinical - Persistent bleeding or rebleeding after endoscopic therapy
evidence of a GI bleed Indication for repeat endoscopic treatment
should receive an
endoscopy within 24 RISK-STRATIFICATION TOOLS FOR UPPER GASTROINTESTINAL
hours, and while awaiting HEMORRHAGE
the EGD, they should be A. BLATCHFORD SCORE
treated with a PPI. AT PRESENTATION POINTS
• After the index endoscopy, Systolic blood pressure
treatment strategies 100–109 mmHg 1
depend on the 90–99 mmHg 2
appearance of the lesion <90 mmHg 3
at endoscopy. Blood urea nitrogen
• Endoscopic therapy is 6.5–7.9 mmol/L 2
instituted If bleeding is 8.0–9.9 mmol/L 3
active or, when bleeding 10.0–24.9 mmol/L 4
has already stopped, if ≥25 mmol/L 6
there is a significant risk of Hemoglobin for men
re-bleeding. The ability to 12.0–12.9 g/dL 1
predict the risk of re-
10.0–11.9 g/dL 3
bleeding permits
<10.0 g/dL 6
prophylactic therapy,
Hemoglobin for women
closer monitoring, and earlier detection of hemorrhage in high-risk
patients. 10.0–11.9 g/dL 1
• The Forrest Classification was developed in an attempt to assess <10.0 g/dL 6
this risk on the basis of endoscopic findings and to stratify the Other variables at presentation
patients into low-, intermediate-, and high-risk: Pulse ≥100 beats/min 1
o Forrest I-IIa: endoscopic therapy is recommended in cases of Melena 1
active bleeding as well as for a visible vessel Syncope 2
o Forrest IIb: cases of an adherent clot; the clot is removed and Hepatic disease 2
the underlying lesion is evaluated; typically seen Cardiac failure 2
o when there is spontaneous stopping of bleeding B. ROCKALL SCORE
o Forrest IIb, IIc, and III: typically, can be managed VARIABLE PTS
o medically i.e. PPIs and test the presence of H. Pylori Age
o Forrest Ia, Ib, and IIa: warrants surgical intervention <60 y 0
The Forest Classification for Endoscopic Findings and Rebleeding 60–79 y 1
Risks in Peptic Ulcer Disease ≥80 y 2
CLASSIFICATION REBLEEDING RISK Shock
Grade Ia active, pulsatile bleeding High CLINICAL Heart rate >100 beats/min 1
ROCKALL
Grade Ib active, non-pulsatile bleeding High SCORE Systolic blood pressure <100 2
Grade IIa non-bleeding visible vessel High mmHg
Grade IIb adherent clot Intermediate Coexisting illness
Grade IIc ulcer with black spot Low Ischemic heart disease, congestive heart 2
COMPLETE
Grade III clean, non-bleeding ulcer bed Low failure, other major illness
ROCKALL
SCORE Renal failure, hepatic failure, metastatic 3
cancer
MALLORY – WEISS SYNDROME
Endoscopic diagnosis
- Acute upper GI bleeding following vomiting, is considered to be the No lesions observed, Mallory-Weiss 0
cause of up to 15% of all severe upper GI bleeds. syndrome Peptic ulcer, erosive disease, 1
- Mechanism is similar to spontaneous esophageal perforation: an esophagitis Cancer of the upper GI tract 2
acute increase in intra-abdominal pressure against a closed glottis in
Endoscopic stigmata of recent
a patient with a hiatal hernia.
hemorrhage 0
o Vomiting is not an obligatory factor, as there may be other
Clean base ulcer, flat pigmented spot 2
causes of an acute increase in intra-abdominal pressure, such
Blood in upper GI tract, active bleeding,
as paroxysmal coughing, seizures, and retching
visible vessel, clot
- Mucosal tears in the GEJ (gastroesophageal junction)
- Can also follow from paroxysmal coughing, retching and seizures

2⏐6
liv
MANAGEMENT Algorithm for the treatment of bleeding peptic ulcer.
SURGERY B
- PPIs are the mainstay of medical therapy for PUD, but high dose
H2RAs and sucralfate are also quite effective. Patients hospitalized Hospital admission Bleeding peptic ulcer

for ulcer complications should receive PPI by continuous IV infusion


and, when discharged, should be considered for life- long PPIs
unless the definitive cause is eliminated or a definitive operation Resuscitate
Continuous IV PPI drip
performed. EGD
- If H. pylori infection is documented, it should be treated with one of
several acceptable regimens
- Long-term maintenance PPI therapy should be considered in all 20% high risk 80% Low risk
patients admitted to hospital with ulcer complications, all high-risk
patients on NSAIDs or aspirin (the elderly or debilitated), and all
patients with a history of recurrent ulcer or bleeding Yes Shock? No

Yes Transfusion? No
Treatment regimens for Helicobacter pylori Yes Active bleeding on EGD? No
MEDICATIONS/DOSE/FREqUENCy DURATION
Yes Visible vessel on EGD? No
PPI + clarithromycin 500 mg bid + amoxicillin 1000 mg bid 10–14 d
Yes Abnormal PT, PTT, or platelets? No
PPI + clarithromycin 500 bid + metronidazole 500 bid 10–14 d
PPI + amoxicillin 1000 mg bid, then 5d
PPI + clarithromycin 500 mg bid + tinidazole 500 mg bid 5d Endoscopic hemostatic Rx
Bleeding stops
Salvage regimens for patients who fail one of the above initial Consult surgeon

regimens:
Bismuth subsalicylate 525 mg qid + metronidazole 250 10–14 d Bleeding recurs Lifelong PPI
mg qid + tetracycline 500 mg qid + PPI in hospital Test + Rx H. pylori
Avoid NSAIDs/ASA if possible
PPI + amoxicillin 1000 mg bid + levofloxacin 500 mg daily 10 d
PPI = proton pump inhibitor. Source: Data from Chey et al.63
Bleeding persists
>4 PRBC transfused/24h
SURGICAL INTERVENTION Deep ulcer eroding big vessel Discharge
Hemodynamic instability
- Criteria for Bleeding Peptic Ulcer Surgical Intervention: Hemostatic Rx unavailable
o 2 endoscopic failures
o Elderly patients and patients with multiple comorbidities Bleeding recurs

o Deep bleeding ulcers on the posterior bulb or lesser gastric O.R.


curvature
- Indications for Immediate Surgery in GI Bleeding: All Algorithm
Figure 26-42. patientsforadmitted
the treatmentto hospital
of bleeding with
peptic bleeding
ulcer. peptic ulcer
ASA = acetylsalicylic should
acid; EGD be
= esophagogastroduodenoscopy;
o Hemodynamic instability despite vigorous resuscitation (>6- O.R. = operating room; PPI =resuscitated
adequately proton pump inhibitor;
andPRBC = unit of
started onpacked red blood cells;
continuous PT = prothrombin
IV PPI.72 time; PTT = partial throm-
Seventy-five
boplastin time; Rx = treatment.
unit transfusion) percent of patients will stop bleeding with these measures alone, but 25%
o Failure of endoscopic techniques to arrest hemorrhage will continue to bleed or will rebleed in hospital. It is important to identify
o Recurrent hemorrhage after initial stabilization (with up to this high-risk
The standard group
operation for earlyPUD
obstructing with clinical andbecause
is vagotomy endoscopic parameters
of intractable PUD should raisebecause,
red flags for the sur-
two attempts at obtaining endoscopic hemostasis) and antrectomy. Alternatively vagotomy and gastrojejunostomy
essentially, all the deaths from bleeding geon: Maybe the patient has a missed cancer; maybe the patient
ulcer occur in this group. Surgical
should be considered if a difficult duodenal stump is anticipated is noncompliant (not taking prescribed PPI, still taking NSAIDs,
o Shock associated with recurrent hemorrhage consultation
with resection. is mandatory,
HSV and gastrojejunostomy may and endoscopic
be compara- still hemostatic
smoking); maybetherapy
the patient(cautery,
has Helicobacter despite the
o Continued slow bleeding with a transfusion requirement ble to V+Aepinephrine injection,
for obstructing ulcer disease, clipping)
76
and sometimesis indicated
has andofusually
presence successful
a negative in these
test or previous treatment. Because
exceeding 3 units/day appeal because it can be done laparoscopically, and because
high-risk patients.73,74 Indications for operation acid secretion can be totally blocked and H. pylori eradicated
include massive
it does not complicate future resection, if needed. However, with modern medication, the question remains: “Why does the
potentially hemorrhage
curable gastric orunresponsive
duodenal cancers canto endoscopic
be missed control,
patient have and transfusion
a persistent ulcer diathesis?” The surgeon should
SURGICAL TECHNIQUE: requirement of more than four to six unitsreview
with this approach. of blood, despite
the differential attempts
diagnosis at ulcer before any
of nonhealing
- Suture ligation of the bleeder endoscopic control. Lack consideration
of availability of a therapeutic of operative treatment (Table 26-13).
endoscopist,
Intractable or Nonhealing Peptic Ulcer Surgical treatment should be considered in patients with
- Suture ligation and non - resective ulcer operation (HSV or V + D) Intractabilityrecurrent hemorrhage
should be an unusual indicationafter one or
for peptic more nonhealing
ulcer attemptsor at endoscopic
intractable PUD whocontrol,
have multiple recurrences,
- Gastric resection (vagotomy and ulcer resection) lack of The
operation nowadays. availability offor
patient referred blood
surgicalfor transfusion,
evaluation largerepeat
ulcers (>2hospitalization for
cm), complications (obstruction, perforation, or
o Gastric ulcer requires biopsy if not resected. bleeding ulcer, and con- current indications for surgery such as perforation
or obstruction, are also indications for surgery. Patients with massive
Surgical options in the treatment of duodenal and gastric ulcer bleeding from high-risk lesions (e.g., posterior duodenal ulcer with erosion
INDICATION DUODENAL GASTRIC of gastroduodenalartery, or lesser curvature gastric ulcer with erosion of
Bleeding 1. Oversewa 1. Oversew and biopsya left gastric artery or branch) should be considered for early operation. Early
2. Oversew, V + D 2. Oversew, biopsy, V + D operation should also be considered in patients more than 60 years of age,
3. V + A 3. Distal gastrectomyb those presenting in shock, those requiring more than four units of blood in
Perforation 1. Patcha 1. Biopsy and patcha 24 hours or eight units of blood in 48 hours, those with rebleeding, and
2. Patch, HSV 2. Wedge excision, V + D those with ulcers >2 cm in diameter. The mortality rate for surgery for
3. Patch, V + D 3. Distal gastrectomyb bleeding peptic ulcer is around 20%. Angiography and embolization may
Obstruction 1. HSV + GJ 1. Biopsy; HSV + GJ be useful in some patients.
2. V + A 2. Distal gastrectomyb
Intractability/ 1. HSVb 1. HSV and wedge
nonhealing 2. V + D 3. V + A excision
2. Distal gastrectomy
a Unless the patient is in shock or moribund, a definitive procedure should be
considered.
b Operation of choice in low-risk patient.
GJ = gastrojejunostomy; HSV = highly selective vagotomy; V + A = vagotomy and
antrectomy; V + D = vagotomy and drainage.

LOWER GI BLEEDING
MECKEL’S DIVERTICULUM
- Most common congenital anomaly of the GIT
- TRUE Diverticula - their walls contain all of the layers found in
normal small intestine
- 60% contain heterotopic mucosa
3⏐6
liv
o over 60% consist of gastric mucosa -
SURGERY B
Rarely diagnosed prior to surgical intervention
o Pancreatic acini (next most common) - Incidental finding during endoscopy or radiology
o Others: Brunner’s glands, pancreatic islets, colonic mucosa, - Enteroclysis - accuracy of 75% but is usually not applicable during
endometriosis, and hepatobiliary tissues acute presentations of complications related to Meckel’s diverticula
- Radionuclide scans - helpful in the diagnosis of Meckel’s
“RULE OF TWOS” diverticulum
- 2% prevalence o this test is positive only when the diverticulum contains
- 2:1 MALE predominance associated ectopic gastric mucosa that is capable of uptake
- 2 feet from ileocecal valve (Location) of the tracer
o usually found in the ileum within 100 cm of the ileo- cecal o accuracy
valve § 90% in pediatric pxs
- Half are asymptomatic under 2 yrs. old § less than 50% in adults

TREATMENT
- Recommended: diverticulectomy with removal of associated bands
connecting the diverticulum to the abdominal wall or intestinal
mesentery
- For bleeding, segmental resection of ileum that includes both the
diverticulum and the adjacent ileal peptic ulcer should be
performed
o may also be necessary if the diverticulum contains a tumor or
if the base of the diverticulum is inflamed or perforated
PATHOPHYSIOLOGY - Management of incidentally found Meckel’s is controversial
- During the eighth week of gestation, the omphalomesenteric
(vitelline) duct normally undergoes obliteration. ACQUIRED DIVERTICULA
- Failure or incomplete obliteration of the vitelline duct à Most
- False diverticula à their walls consist of mucosa and submucosa
commonly, Meckel’s Diverticulum
but lack a complete muscularis
- Other abnormalities
- Asymptomatic unless complications arise
o Formation of omphalomesenteric fistula
- More common in the duodenum near the ampulla à known as
o Enterocyst
periampullary, juxtapapillary, or perivaterian diverticula
o Fibrous band connecting the intestine to the umbilicus
- Prevalence
- Bleeding associated with Meckel’s diverticulum à usually the
o duodenal diverticula: 0.16% - 6%
result of ileal mucosal ulceration that occurs adjacent to acid-
§ rare in patients under 40 years.
producing, heterotopic gastric mucosa located within the
§ mean age of diagnosis: 56 - 76 years
diverticulum.
o jejunoileal diverticula: 1% to 5%.65
- Meckel’s diverticula can be found in inguinal or femoral hernia sacs
§ prevalence increased w/age; sixth and seventh
(known as Littre’s hernia). These hernias, when incarcerated, can
decades of life
cause intestinal obstruction.
- Jejunoileal diverticula
o 85% jejunum
CLINICAL PRESENTATION
o 15% ileum
- Asymptomatic unless complications arise
o 5% both
- Lifetime incidence rate of complications is 4% to 6%
- Diverticula in the jejunum tend to be large and accompanied by
- MOST common presentation
multiple other diverticula
o Bleeding (Most common in children less than 18yrs)
- Those in the ileum tend to be small and solitary
§ rare among patients older than 30 yrs
- Pathophysiology:
o Intestinal Obstruction (Adults)
o hypothesized to be related to acquired abnormalities of
o Diverticulitis (20% symptomatic patients)
intestinal smooth muscle or dysregulated motility à herniation
§ associated with a clinical syndrome that is
of mucosa and submucosa through weakened areas of
indistinguishable from acute appendicitis
muscularis
- Carcinoid tumors present in 0.5% to 3.2% of resected diverticuli
o can lead to bacterial overgrowth, leading to vitamin B12
- Common neoplasm seen
deficiency, megaloblastic anemia, malabsorption, and
steatorrhea

OBSCURE GI BLEEDING
- GI Bleeding with no source identified
o Overt GI bleeding - presence of hematemesis, melena, or
hematochezia.
o Occult GI bleeding - occurs in the absence of overt bleeding
and is identified on laboratory tests (e.g., iron-deficiency
anemia) or examination of the stool (e.g., positive guaiac
Causes of Obstruction: test)
1. Volvulus of the intestine around the fibrous band attaching the - Occult in 20% of cases
diverticulum to the umbilicus - Small intestinal angiodysplasia account for approximately 75% of
2. Entrapment of intestine by a mesodiverticular cases in adults
3. Intussusception with the diverticulum acting as a lead point - Neoplasms account for 10% of cases
4. Stricture secondary to chronic diverticulitis - Meckel’s diverticulum MOST COMMON in children
- Other Sources
o Chron’s disease
o NSAID induced ulcers and erosions
o Infectious enterides
o Vasculitis, Ischemia
o Intussusception

DIAGNOSIS

4⏐6
liv
Diagnostic and management algorithm for obscure gastrointestinal return of bile suggests that the source of bleeding is distal to
o
SURGERY B
(GI) bleeding. the ligament of Treitz.
- 1169If aspiration reveals blood or non- bile secretions, or if symptoms
Obscure gastrointestinal bleeding
suggest an upper intestinal source, esophagogastroduodenoscopy
is performed.
Rule out upper and lower GI - Anoscopy and/or limited proctoscopy - can identify hemorrhoidal
bleeding;
EGD and colonoscopy bleeding.
- Technetium-99 (99mTc)-tagged red blood cell (RBC) scan -
Minor bleeding Major bleeding extremely sensitive and is able to detect as little as 0.1 mL/h of
bleeding; however, localization is imprecise.

CHAPTER 28 SMALL INTESTINE


(intermittent) (persistent)

o Positive: angiography can then be both diagnostic and


Small bowel Stable Unstable potentially therapeutic.
series Negative
Enteroclysis and - If the patient is hemodynamically stable, a rapid bowel preparation
Enteroscopy
Capsule
patient stable
Tagged Operating
(over 4–6 hours) can be performed to allow colonoscopy.
endoscopy RBC scan room - Colonoscopy – may identify the cause of the bleeding
Positive Negative Positive o cautery or injection of epinephrine into the bleeding site may be
Initiate appropriate
Repeat EGD/
Angiography Source Source of
used to control hemorrhage.
Colonoscopy if
therapy
rebleeds
and treatment uncertain bleeding identified - Colectomy - may be required if bleeding persists despite
interventions.
Localize bleeding: Treat source eg.
- Intraoperative colonoscopy and/or enteroscopy - may assist in
Serial clamping or intraoperative small bowel localizing bleeding.
enteroscopy followed by resection resection
- If colectomy is required, a segmental resection is preferred if the
-
Figure 28-29. Enteroscopy
Diagnostic is playing
and management anforincreasingly
algorithm important
obscure gastrointestinal role.
(GI) bleeding. EGD = esophagogastroduodenoscopy; bleeding source can be localized.
RBC = red
- bloodSeveral
cell. endoscopic techniques for visualizing the small intestine: - “Blind” subtotal colectomy - may very rarely be required in a patient
o push enteroscopy,
injury, Meckel’s and acquired diverticula, neoplasms (especially fever and tachycardia. Plain abdominal radiographs may reveal who is hemodynamically unstable w/ ongoing colonic hemorrhage of
o
lymphoma, adenocarcinoma, Sonde enteroscopy,
and melanoma), and foreign bodies. free intraperitoneal air if intraperitoneal perforation has occurred. an unknown source.
Among iatrogenic injuries, duodenal perforation dur- If perforation is suspected but not clinically obvious, CT scan-
o intraoperative enteroscopy,
ing ERCP with endoscopic sphincterotomy (ES) is the most ning should be performed. Jejunal and ileal perforations require
o prior to proceeding, it is crucial to irrigate the rectosigmoid
o
common. This complication double-
occurs inballoon
0.3% to 2.1% endoscopy,
of cases. surgical repair or segmental resection. o re-examine the mucosa of the anal canal and rectum by
Patients who have o undergone
wirelessBillroth II gastrectomy
capsule are at
enteroscopy anoscopy and proctoscopy to ensure the source of ongoing
increased risk of duodenal perforations as well as free jejunal Chylous Ascites
Push
perforations enteroscopy
during ERCP. Although ERCP-related duodenal Chylous ascites refers to the accumulation of triglyceride-rich bleeding is not distal to the planned resection margin).
- canadvancing
perforations a long endoscope
result in a free perforation, most are retroperi-(suchperitoneal
as a pediatric
fluid with aor adult
milky or creamy appearance, caused - Occult blood loss from GIT à may manifest as iron-deficiency
toneal. Manifestations of such contained duodenal perforations by the presence of intestinal lymph in the peritoneal cavity.
following ERCPcolonoscope
can resemble thoseor a specialized
of ERCP-induced pancre-instrument) beyond
Chylomicrons, produced the ligament
by the intestine andof
secreted into lymph anemia or may be detected with FOBT.
atitis, including Treitz into the proximal jejunum
hyperamylasemia. during the absorption of long-chain fatty acids, account for the o Because colon neoplasms bleed intermittently and rarely
CT scanning is the most sensitive test for diagnosing duo- characteristic appearance and triglyceride content of chyle.
- allow for visualization of approximately
denal perforations; positive findings include pneumoperitoneum
60 cm of the proximal jejunum
The most common etiologies of chylous ascites in Western
present with rapid hemorrhage, the presence of occult fecal
for free-perforations,
Diagnostic yield: 3%
but more commonly to 65%. air,
retroperitoneal countries are abdominal malignancies and cirrhosis. In Eastern blood should always prompt a colonoscopy.
contrast-extravasation,
Alsp and paraduodenal
allows fluid collections. If of
for cauterization and developing
all bleeding sites.countries, infectious etiologies, such as tuber- o Unexplained iron-deficiency anemia is also an indication for
patients undergoing a therapeutic ERCP are imaged with a CT culosis and filariasis, account for most cases. Chylous ascites
scan following the procedure, up to 30% will have evidence of can also develop as a complication of abdominal and thoracic colonoscopy.
Sonde
air in the enteroscopy
retroperitoneum, but the majority are asymptomatic. operations and trauma. Operations particularly associated with - Hematochezia is commonly caused by hemorrhoids or a fissure.
These patients do not require any specific therapy.70 this complication include abdominal aortic aneurysm repair,
- casesaoflong,
True thin fiberoptic
retroperitoneal perforations ofinstrument
the duode- is retroperitoneal
propelledlymph through the intestine
node dissection, bycava resec-
inferior vena o Sharp, knife-like pain and bright red rectal bleeding with bowel
peristalsis
num can be managed following
nonoperatively, inflation
in the absence of a balloon
of progres- at transplantation.
tion, and liver the instrument’s tip of chylous asci-
Other etiologies movements suggest the diagnosis of fissure.
sion and sepsis. However, intraperitoneal duodenal perforations tes include congenital lymphatic abnormalities (e.g., primary
- Visualization is done during instrument
require surgical repair with pyloric exclusion and gastrojejunos-
withdrawal; approximately
lymphatic hypoplasia), radiation, pancreatitis, and right-sided
o Painless, bright red rectal bleeding with bowel movements is
50% to 75%
tomy or tube duodenostomy. of the
Iatrogenic small-intestinal
small bowel perfora- mucosa
heart failure.can be examined. often secondary to a friable internal hemorrhoid that is easily
tion incurred
- during endoscopy, if immediately recognized, can
Limitations: Three mechanisms have been postulated to cause chylous detected by anoscopy.
sometimes be repaired using endoscopic techniques. ascites: (a) exudation of chyle from dilated lymphatics on the
Perforation o Lacksandbiopsy
of the jejunum or therapeutic
ileum occurs into the peri- capability
wall of the bowel and in the mesentery caused by obstruction o In the absence of a painful, obvious fissure, any patient with
toneal cavity and o Lacksovert
usually causes tipsymptoms
deflection capability,
and signs, such limitingvessels
of lymphatic complete
at the basemucosal
of the mesentery or the cisterna rectal bleeding should undergo a careful digital rectal
as abdominal pain, tenderness, and distention accompanied by chili (e.g., by malignancies); (b) direct leakage of chyle through
visualization examination, anoscopy, and proctosigmoidoscopy.
o Failure to diagnose a source in the distal anorectum should
Wireless capsule enteroscopy prompt colonoscopy.
1184
- Relies on a radiotelemetry capsule enteroscope that is small enough Acute colonic bleeding

to swallow and has no external wires, fiberoptic bundles, or cables


- While the capsule is being propelled through the intestine by Volume resuscitation
plus blood transfusion

peristalsis, video images are transmitted using radiotelemetry to an NG aspirate


NG aspirate
array of detectors attached to the patient’s body. negative positive

o detectors capture the images and permit continuous Proctoscopy Gastroduodenoscopy

triangulation of the capsule location in the abdomen, Rule out anorectal bleeding + endoscopic treatment
PART
UNIT II

Bleeding stopped or slowed down Massive life-threatening bleeding


facilitating the localization of lesions detected Elective colonoscopy Mesenteric arteriography

Positive Negative Positive Negative

Capsule endoscopy Endoscopic


treatment
Observe Moderate bleeding
continued
Vasopressin
or emboli
Explore, intraoperative
endoscopy
SPECIFIC CONSIDERATIONS

- For patients who are hemodynamically stable but continues to bleed. Rebleeding
Rebleeding
Fail Positive Negative

- Success rates as high as 90% in identifying a small bowel pathology. Segmental


resection
See moderate bleeding
or massive bleeding
Segmental
resection
Segmental
resection
Total
colectomy

- The inability to perform biopsies or carry out any therapeutic


or
interventions of capsule endoscope likely prevents the improved 99MTc RBC scintigraphy Urgent colonoscopy

diagnostic yield of the test Positive Negative


Positive Negative

Mesenteric arteriography Explore, intraoperative Colonoscopic treatment Explore, intraoperative


endoscopy or explore, segmental endoscopy
Positive Negative resection
LOWER GI BLEEDING Vasopressin Explore, intraoperative
Positive Negative Positive Negative
Segmental Total Segmental Total
- The first goal in evaluating and treating a patient with gastrointestinal or emboli
Fail
endoscopy
Positive Negative
resection colectomy resection colectomy

hemorrhage is adequate resuscitation. Segmental Segmental Total


resection resection colectomy
o ensuring a patent airway
o supporting ventilation, Algorithm for treatment
Figure 29-7. Algorithm of colorectal
for treatment of colorectal hemorrhage. NG hemorrhage. (Schwartz)
= nasogastric; Tc = technetium-99; RBC = red blood cell. 99m

o optimizing hemodynamic parameters apply, (Reproduced with permission of Taylor & Francis, LLC from Gordon PH, Nivatvongs S, eds. Principles and Practice of Surgery for the
For
Colon,patients in whom
Rectum, and Anus. 2nd ed. Newbleeding from
York: Marcel Dekker, Inc.;an obscure
1999:1279. GIconveyed
Permission source has
through apparently
Copyright Clearance Center,
o coagulopathy and/or thrombocytopenia should be corrected stopped,
Inc.)
push enteroscopy or capsule enteroscopy is a reasonable initial
- The second goal is to identify the source of hemorrhage. study.
movements, Ifhard
thesestools,examinations do
or excessive straining. A nothis-reveal
careful a potential
(colonic source
inertia) refractory to maximalofmedical
bleeding,
interventions.
tory of these symptoms often clarifies the nature of the problem. While this operation almost always increases bowel movement
- Most common source of GIT hemorrhage: esophageal, gastric, or thenConstipation
enteroclysis has manyshould be performed.
causes. Underlying metabolic, Standard small
frequency, complaints bowel
of diarrhea, follow-
incontinence, and abdominal
duodenal through
pharmacologic, examinations are associated
endocrine, psychological, and neurologic with a low
pain are diagnostic
not infrequent, and patientsyield
should bein this selected
carefully
causes often contribute to the problem. A stricture or mass and counseled. 15

o nasogastric aspiration should always be performed lesion should be excluded by colonoscopy, barium enema, or
CT colonography. After these causes have been excluded, eval- Diarrhea and Irritable Bowel Syndrome. Diarrhea is also
5⏐6 uation focuses on differentiating slow-transit constipation from a common complaint and is usually a self-limited symptom of
liv outlet obstruction. Transit studies, in which radiopaque markers
are swallowed and then followed radiographically, are useful for
infectious gastroenteritis. If diarrhea is chronic or is accompa-
nied by bleeding or abdominal pain, further investigation is
diagnosing slow-transit constipation. Anorectal manometry and warranted. Bloody diarrhea and pain are characteristic of colitis;
setting and should be avoided. If still no diagnosis has been made, a
SURGERY B
DIVERTICULAR DISEASE
“watch-and-wait” approach is reasonable, although angiography should be - Result from erosion of the peridiverticular arteriole
considered if the prior episode of bleeding was overt. Angiography can - Most significant in elderly patients
reveal angiodysplasia and vascular tumors in the small intestine even in the - The exact bleeding source may be difficult to identify
absence of ongoing bleeding. - Only 3% - 15% of individuals experience any bleeding (Sabiston, 20th
For persistent mild bleeding from an obscure GI source, push and capsule ed.)
enteroscopy can be used. If these examinations are nondiagnostic, then - 80% of patients, bleeding stops spontaneously
99m Tc-labeled RBC scanning should be performed and, if positive, o 10% will rebleed in 1 yr (Sabiston)
followed by angiography to localize the source of bleeding. 99m Tc- o 50% will rebleed in 10 yrs (Sabiston)
pertechnetate scintigraphy to diagnose Meckel’s diverticulum should be - Symptomatic diverticula
considered, although its yield in patients older than 40 years of age is
extremely low. Patients who remain undiagnosed but continue to bleed and Management:
those with recurrent episodic bleeding significant enough to require blood - Resuscitation
transfusions should then undergo exploration with intraoperative - Localization of bleeding:
enteroscopy. o Colonoscopy - may occasionally identify a bleeding
Patients with persistent severe bleeding from an obscure source should diverticulum that may then be treated with
undergo angiography to help localize the bleeding source. Therapy can be epinephrine injection or cautery.
tailored based on the source. Push enter- oscopy can also be attempted, o Angiography - may be diagnostic and therapeutic
but capsule enteroscopy is too slow to be applicable in this setting. If these o Endoscopic clips
examinations fail to localize the source of bleeding, exploratory
laparoscopy or laparotomy with intraoperative enteroscopy is indicated.
Intra- operative enteroscopy can be done during either laparotomy or
laparoscopy. An endoscope (usually a colonoscope) is inserted into the
small bowel through peroral intubation or through an enterotomy made in
the small bowel or cecum. The endoscope is advanced by successively
telescoping short segments of intes- tine onto the end to the instrument. In
addition to the endoscopic image, the transilluminated bowel should be
examined externally with the operating room lights dimmed, as this
maneuver may facilitate the identification of angiodysplasias. Identified
lesions should be marked with a suture placed on the serosal surface of the § Giant Colonic Diverticulum
bowel; these lesions can be resected after completion of endoscopy. - Extremely RARE
Examination should be performed during instrument insertion rather than - Most occur on: Anti mesenteric side of the sigmoid colon
withdrawal because instrument- induced mucosal trauma can be confused - Asymptomatic or may present with vague abdominal complaints
with angiodysplasias. such as pain, nausea, or constipation
- Barium enema usually diagnostic
o Plain radiographs may suggest the diagnosis
- Resection of the involved colon and diverticulum is
recommended
- Complications: Perforation, volvulus, obstruction

§ Right Sided Diverticula


- Less common
- Occur more often in younger patients than do left-sided
diverticula and are more common in people of Asian descent
than in other populations
- Most patients with right-sided diverticula are asymptomatic
- May mimic acute appendicitis
o Diagnosis of right-sided diverticulitis is subsequently
made in the operating room
- Diverticulectomy may be done (If there is a single large
diverticu- lum and minimal inflammation) but ileocecal
resection is preferred
- Hemorrhage rarely occurs and should be treated in the same
fashion as hemorrhage from a left-sided diverticulum.

Sources:
- 2021 PPT
- Schwartzs Principles of Surgery, 10th Ed
- Sabiston textbook of Surgery 20th ed.
- Kimi ledda trans <3
Sabiston 20th ed.

RBC SCINTIGRAPHY

- The bleeding site is in the descending colon.

6⏐6
liv

You might also like